3
$\begingroup$

Consider an ARMA process expressed in lag operator notation, $$ \Phi(L)x_t=\theta(L)\varepsilon_t. $$ Let $\text{Var}(\varepsilon_t)=\sigma^2_{\varepsilon}$.

Question: Can the long-run variance of $x_t$ be lower than the error variance? If so, could you provide an example?

A related thread with an answer that may help answer my question is "Long-run variance of ARMA(p,q)".
A related new question is "Can unconditional variance of an ARMA process be lower than its error variance?".

$\endgroup$
2
  • $\begingroup$ The second link is the same as the first one $\endgroup$
    – gunes
    Feb 2, 2021 at 9:08
  • $\begingroup$ @gunes, good catch! Now fixed. $\endgroup$ Feb 2, 2021 at 9:09

1 Answer 1

3
$\begingroup$

Let's denote the long-run variance as $\operatorname{var}_L(y_t)$ to prevent confusion. From the formula provided in the linked post, a proper choice of coefficients can lead to smaller long-run variance. Let our process be defined as $$y_t=e_t-0.6e_{t-1}$$ then, the long-run variance will be $$\operatorname{var}_L(y_t)=\sigma^2 \left(\frac{1-0.6}{1}\right)^2=0.16\sigma^2<\sigma^2$$ Note that the missing definiton of coefficients in that post is ($b_0=1$) $$y_t=\sum_{i=1}^p a_py_{t-p}+\sum_{i=0}^q b_qe_{t-q}$$

$\endgroup$
3
  • $\begingroup$ I believe the formula there is wrong. Your example yields $\text{Var}(x_t)=1.36\sigma^2_\varepsilon>\sigma^2_\varepsilon$. Here is a quick simulation to show that: n=1e6; set.seed(1); e=rnorm(n); x=arima.sim(model=list(ma1=-0.6),n=n,innov=e); var(e); var(x). $\endgroup$ Feb 2, 2021 at 6:05
  • 1
    $\begingroup$ @RichardHardy But, long-term variance is not $var(x_t)=\gamma(0)$, where $\gamma(t)$ is the auto covariance function. We also have $\gamma(1)=\gamma(-1)=-0.6\sigma^2$ and higher order covariances $0$, which yields $$\operatorname{var}_L(y_t)=\sum_{k=-\infty}^\infty \gamma(k) = 0.16\sigma^2$$ $\endgroup$
    – gunes
    Feb 2, 2021 at 8:38
  • $\begingroup$ Ahh, I forgot what the term long-run variance actually means. I meant to ask about the unconditional variance of $x$ instead, which I actually did by writing $\text{Var}(x_t)$. I will edit the question to clarify. Or better yet, I will post a new question so that your answer gets the credit it deserves. ... And I did: here is my new question. $\endgroup$ Feb 2, 2021 at 9:04

Your Answer

By clicking “Post Your Answer”, you agree to our terms of service and acknowledge you have read our privacy policy.

Not the answer you're looking for? Browse other questions tagged or ask your own question.